LSAT and Law School Admissions Forum

Get expert LSAT preparation and law school admissions advice from PowerScore Test Preparation.

 Administrator
PowerScore Staff
  • PowerScore Staff
  • Posts: 8917
  • Joined: Feb 02, 2011
|
#41449
Please post your questions below!
 fersian
  • Posts: 19
  • Joined: Jan 19, 2019
|
#61959
Hello,
Can you please explain how 'E' is wrong?
 Adam Tyson
PowerScore Staff
  • PowerScore Staff
  • Posts: 5153
  • Joined: Apr 14, 2011
|
#61969
Answer E makes the paradox here even worse, fersian, for if the restaurants in the business district are already competing with malls that offer free parking, and the parking rates for the business district go up, that gives the malls a bigger competitive edge. Why, then, did the restaurants in the business district increase sales? Shouldn't they have lost business to the malls?

Since answer E does not explain why the sales increased at restaurants in the business district, it fails to resolve the paradox. Answer D - higher turnover - does, because it could mean customers cycling through the restaurants faster and getting more people in and out in the same amount of time. Move 'em through!
 fersian
  • Posts: 19
  • Joined: Jan 19, 2019
|
#62045
Thank you for the explanation!

Get the most out of your LSAT Prep Plus subscription.

Analyze and track your performance with our Testing and Analytics Package.